Medical-Surgical Nursing Exam 6

Practice Mode

Welcome to your Medical-Surgical Nursing Exam 6! This exam is carefully curated to help you consolidate your knowledge and gain deeper understanding on the topic.

 

Exam Details

  • Number of Questions: 50 items
  • Mode: Practice Mode

Exam Instructions

  1. Practice Mode: This mode aims to facilitate effective learning and review.
  2. Instant Feedback: After each question, the correct answer along with an explanation will be revealed. This is to help you understand the reasoning behind the correct answer, helping to reinforce your learning.
  3. Time Limit: There is no time limit for this exam. Take your time to understand each question and the corresponding choices.

Tips For Success

  • Read each question carefully. Take your time and don't rush.
  • Understand the rationale behind each answer. This will not only help you during this exam, but also assist in reinforcing your learning.
  • Don't be discouraged by incorrect answers. Use them as an opportunity to learn and improve.
  • Take breaks if you need them. It's not a race, and your understanding is what's most important.
  • Keep a positive attitude and believe in your ability to succeed.

Remember, this exam is not just a test of your knowledge, but also an opportunity to enhance your understanding and skills. Enjoy the learning journey!

 

Click 'Start Exam' when you're ready to begin. Best of luck!

💡 Hint

Pemphigus vulgaris is a rare autoimmune disorder that primarily affects the skin and mucous membranes. It is characterized by the formation of certain types of symptoms on the skin's surface.

1 / 50

1. What is the primary complication that a nurse must assist a patient in managing when they have been diagnosed with pemphigus vulgaris?

💡 Hint

Consider what a hydrocele is - an accumulation of serous fluid in a sac-like cavity, especially in the scrotal sac. How might the body typically respond to such an accumulation of fluid?

2 / 50

2. A nursing assistant at a conference learns that one of her patients has been diagnosed with a hydrocele. When she inquires about the standard treatment for this condition, what would be the most appropriate response?

💡 Hint

Informed consent involves the patient's understanding and voluntary agreement to a medical procedure or treatment. This process requires the patient to be in a clear state of mind. Reflect on what could potentially impair a patient's ability to provide valid consent.

3 / 50

3. In the context of a legal framework, under which circumstance is a patient scheduled for a below-the-knee amputation of the right leg not allowed to sign the operative consent form?

💡 Hint

The motor branch of the facial nerve is responsible for facial expressions. Consider how damage to this nerve might affect the patient's ability to control certain facial movements.

4 / 50

4. After ear surgery, which early symptom might suggest potential damage to the motor branch of the facial nerve?

💡 Hint

After a skin graft procedure, it's important to closely monitor the donor site for signs of infection. One of these signs can be a change in the color and amount of wound drainage.

5 / 50

5. For a patient with full-thickness chest burns who has received a skin graft, what occurrence at the donor site within the first 24 hours should be promptly reported?

💡 Hint

Consider the localized effects of a blood clot.

6 / 50

6. What symptom would suggest to the nurse that Ms. Taylor has suddenly developed a stationary thrombus?

💡 Hint

Reflect on the common early symptom of bladder cancer that might not be associated with pain. It's something that might be noticed during urination.

7 / 50

7. What should the nurse primarily look for when assessing a patient diagnosed with bladder cancer?

💡 Hint

Consider the relationship between hygiene practices, particularly in the genital area, and the risk of infection or other conditions that might lead to cancer.

 

8 / 50

8. While conducting a health behavior class for young men, the nurse discusses potential causes of penile cancer. Which of the following can be identified as a probable cause?

💡 Hint

When discussing sensitive issues such as sexual dysfunction, it is important to approach the topic in a non-judgmental and respectful manner. Focusing on the facts and timeline of the problem can help maintain a professional and comfortable environment for the patient.

9 / 50

9. A female nurse is tasked with obtaining a medical history from a male patient suffering from urinary tract issues and sexual dysfunction. Which statement could potentially make the patient feel more comfortable and willing to discuss his condition?

💡 Hint

Think about the common symptoms often associated with yeast infections, which are caused by the fungus Candida albicans.

10 / 50

10. If Ms. Potter is experiencing Candida albicans-induced vaginitis, which of the following would be a typical sign or symptom?

💡 Hint

Continuous bladder irrigation is often used to prevent clot formation and to ensure clear urine flow following prostate surgery. Think about the appearance of the urine and how the flow rate might affect it.

11 / 50

11. When managing continuous bladder irrigation after prostate surgery, how should the flow rate be adjusted?

💡 Hint

Consider the potential risk if a patient cannot open their mouth properly, particularly in a situation where the body needs to expel something harmful.

12 / 50

12. In patients who have had their fractured mandibles immobilized with wires, what potentially fatal complication can arise postoperatively?

💡 Hint

Reflect on the fact that ovarian cancer is often referred to as a "silent killer." Consider why it might have earned this nickname.

13 / 50

13. While educating a group of young women about ovarian cancer, what would be an accurate statement for the nurse to make?

💡 Hint

The aim of testicular self-examination is to detect abnormalities early. Therefore, it's best performed when the scrotal skin is relaxed, allowing for easier palpation of the testes. Consider what conditions might facilitate this.

14 / 50

14. A nurse is tasked with teaching a patient about testicular self-examination. As part of the instructions, the nurse should mention that the most appropriate time to perform this examination is:

💡 Hint

Consider the actions that could minimize discomfort and avoid further aggravation of the condition.

15 / 50

15. Upon noticing that a patient's knee is swollen and causing discomfort, what nursing measure should be implemented?

💡 Hint

Consider what simple movements could assist in the free flow of fluid within the abdominal cavity during peritoneal dialysis.

16 / 50

16. If, during peritoneal dialysis, the nurse notices that the dialysate drainage from the peritoneal cavity has stopped prematurely, what should the nurse assist the patient in doing?

💡 Hint

Consider the safest and most comfortable method to warm up a patient's feet without causing harm.

17 / 50

17. If Ms. Jones informs the nurse that her feet are cold, which nursing action would be most appropriate?

💡 Hint

Consider the direction that would align with the natural anatomical pathway in this procedure.

18 / 50

18. The nurse is preparing to administer a vaginal suppository to Ms. Davis using a specialized applicator provided with the medication. Which of the following steps is accurate?

💡 Hint

Consider the impact of aging on the body, including the cumulative effects of lifestyle choices, exposure to environmental factors, and the natural aging process itself.

19 / 50

19. In which age group is the prevalence of chronic illness typically the highest?

💡 Hint

Myasthenia gravis is a neuromuscular disorder. The primary issue is with a specific type of receptor at the neuromuscular junction. Consider what this receptor binds to under normal conditions.

20 / 50

20. When a patient diagnosed with myasthenia gravis inquires about the cause of the disease, how should the nurse explain it, given the understanding that the condition is due to:

💡 Hint

The effectiveness of a TENS unit is largely dependent on the individual's pain threshold and comfort level. The device's settings should therefore be adjusted according to the user's feedback.

21 / 50

21. When a nurse is tasked with administering a conventional transcutaneous electrical nerve stimulation (TENS) device to a patient suffering from an inflamed sciatic nerve, which of the following procedures should be followed?

💡 Hint

Reflect on the characteristics of testicular cancer, specifically its propensity to spread, or metastasize, from the original tumor to other parts of the body.

22 / 50

22. In order to appropriately educate men about the significance of testicular self-examination, it's crucial for the nurse to understand that testicular cancer:

💡 Hint

Consider the physiology of sperm production and transport. Even after a vasectomy, some sperm may remain in the upstream portions of the vas deferens and could potentially be released during ejaculation.

23 / 50

23. Mr. Ernest underwent a vasectomy and is wondering why he needs to continue using a birth control method. What would be the most accurate response from the nurse?

💡 Hint

Consider the definition of a cystectomy, which involves removal of all or part of the bladder. Given this, think about the structures most closely associated with the bladder.

24 / 50

24. A patient scheduled for a cystectomy is curious about what the physician will be able to visualize during the procedure. What would be the most accurate response from the nurse?

💡 Hint

Think about the area that a Pap test directly samples from.

25 / 50

25. A nurse is educating young women about health-seeking behaviors, including the importance of the Pap test. What is the primary purpose of this test?

💡 Hint

Consider the anatomical relationship between the site of the surgical incision and the respiratory system, particularly a muscle that plays a key role in breathing.

26 / 50

26. Why should a nurse monitor a patient for respiratory complications after an abdominal cholecystectomy?

💡 Hint

Consider the importance of preventing infection when providing wound care or skin treatments.

27 / 50

27. Following a total abdominal hysterectomy, Ms. Walker experiences a slight increase in temperature and swelling in her right calf. The physician has recommended warm moist compresses for the affected leg. What is the proper nursing action when applying the warm moist compress? The nurse should:

💡 Hint

Scleroderma is a chronic skin disease that often affects other parts of the body. Consider what type of reaction is common in chronic diseases where the body mistakenly attacks its own tissues.

28 / 50

28. While the exact cause of scleroderma remains unknown, what is the prevailing hypothesis regarding its origin?

💡 Hint

When collecting a urine sample from a catheter, the key factors to consider are maintaining sterility to prevent infection and ensuring the most accurate sample possible. Consider the way to collect directly from the source while minimizing contamination.

29 / 50

29. What is the appropriate method for a nurse to obtain a urine sample for ketone testing from a patient's indwelling urinary catheter?

💡 Hint

The sympathetic nervous system plays a significant role in regulating blood pressure. Consider the potential effects of disrupting this system, especially in terms of blood flow and distribution.

30 / 50

30. After a bilateral lumbar sympathectomy, a patient experiences a sudden decrease in blood pressure without any signs of bleeding. What would the nurse identify as the most probable cause?

💡 Hint

Consider the type of solution that would be safest and most physiologically compatible for use in bladder and catheter irrigations.

31 / 50

31. What solution is typically used for continuous or intermittent bladder and catheter irrigations, which the nurse is assigned to monitor in a patient?

💡 Hint

In the diuretic phase of acute renal failure, the kidneys start to recover and begin to excrete large amounts of urine. Consider how this could affect the patient's fluid and electrolyte balance.

32 / 50

32. A week after being diagnosed with acute renal failure, a patient enters the diuretic phase. During this stage, what condition should the patient be meticulously evaluated for?

💡 Hint

Hypocalcemia can be evaluated using certain clinical signs. One of these involves the induction of carpopedal spasm by inflating a blood pressure cuff above systolic pressure, a maneuver associated with a particular clinical sign named after a French physician.

33 / 50

33. While assessing for hypocalcemia, the nurse inflates a blood pressure cuff on the patient's arm, causing carpopedal spasm after approximately 3 minutes. This finding is documented as a positive:

💡 Hint

Consider the position that allows for the most visibility and access during a vaginal examination.

34 / 50

34. A doctor requests that a nurse place a patient in the appropriate position for a vaginal examination. What is the typical position used for this kind of examination?

💡 Hint

Consider the impact of fractured ribs on the mechanics of breathing, especially how the chest wall moves during respiration.

35 / 50

35. What complication might a patient experience as a consequence of fractured ribs?

💡 Hint

Psoriasis is a skin condition known for causing certain distinctive changes to the skin's appearance. Consider what these changes typically look like, particularly in terms of their texture and sheen.

36 / 50

36. What specific symptom should a nurse look for when assessing a patient with psoriasis?

💡 Hint

Consider the actions that could potentially disrupt the healing process after a procedure like electrocauterization.

37 / 50

37. Ms. Anika has undergone electrocauterization of the cervix due to chronic cervicitis. What instructions should the nurse provide her after the procedure?

💡 Hint

Consider the purpose of a sitz bath and the primary area it is meant to treat.

38 / 50

38. A nursing assistant, tasked with giving Ms. Hanna, a patient who has undergone an abdominal hysterectomy, a sitz bath, questions why a regular bathtub can't be used. The most accurate response is based on the fact that a regular bathtub:

💡 Hint

Vincent's Angina, also known as trench mouth, is a severe form of gingivitis. Consider the typical symptoms associated with severe gum disease.

39 / 50

39. What is the primary symptom reported by a patient suffering from Vincent's Angina?

💡 Hint

Reflect on the nature of an ultrasound examination, specifically a transrectal ultrasound (TRUS), which is commonly used for prostate examinations. Consider how the probe is used to generate images of the prostate gland.

40 / 50

40. A patient is slated for a prostate ultrasound examination. To accurately describe the procedure to the patient, the nurse should explain that:

💡 Hint

The main goal of perineal care is to prevent infection. Consider the typical direction of microbial flow in the perineal area when choosing your answer.

41 / 50

41. When performing perineal care for Ms. Paris, in which direction should the nurse gently move the cotton ball or washcloth?

💡 Hint

Psoriasis is a skin condition often characterized by patches of skin that may be itchy, sore, or have a burning sensation. Consider the commonly used class of medications that help to reduce inflammation and slow skin cell turnover.

42 / 50

42. What should the nurse inform a patient with psoriasis regarding the common course of treatment?

💡 Hint

Bladder spasms after prostatectomy can often be triggered by the patient attempting to void against the catheter. Consider which nursing intervention would help to reduce this trigger.

43 / 50

43. Following a prostatectomy, a patient experiences distressing bladder spasms. What action should the nurse take to alleviate these spasms?

💡 Hint

Cholelithiasis is a condition characterized by the presence of gallstones. Consider the role of bile in the digestion of fats and how gallstones might interfere with this process.

44 / 50

44. Why does a patient with cholelithiasis typically experience discomfort following the consumption of fatty foods?

💡 Hint

Psoriasis is typically characterized by a specific type of skin lesion. Think about what these lesions typically look like.

45 / 50

45. Ms. Lora Warren, 27 years old, has been diagnosed with a skin condition and has set an appointment at a clinic that specializes in diagnosing psoriasis. When examining Ms. Lynch's skin for psoriasis, what should the nurse be looking for?

💡 Hint

Consider the proper hygiene practices for an uncircumcised male. Think about how the foreskin acts as a protective layer, and the need for thorough cleansing beneath it.

46 / 50

46. When assigned to provide perineal care for an uncircumcised male patient, which of the following procedures is correct?

💡 Hint

Obstructive jaundice commonly results in changes to the color of body fluids and may cause discomfort in the skin. Consider what changes might occur if bilirubin, a yellow pigment, is not properly eliminated from the body.

47 / 50

47. When evaluating a patient with cholecystitis for the onset of obstructive jaundice, what symptoms would the nurse expect to observe?

💡 Hint

Consider the solutions commonly used for medical cleaning purposes that effectively eliminate bacteria without damaging the material of the tube.

48 / 50

48. When cleaning the inner cannula of a tracheostomy tube, which solution would be the most appropriate for the nurse to use?

💡 Hint

Consider the immediate needs after a procedure that involves penetrating the skin and underlying tissues. The primary concerns are usually related to preventing bleeding and infection.

49 / 50

49. What immediate post-procedure action should a nurse take after a patient has undergone a bone marrow aspiration?

💡 Hint

Consider the steps taken to enhance the efficiency of the examination and the comfort of the client.

50 / 50

50. A nurse is getting a patient ready for a pelvic examination. What instructions would the nurse likely include?

Exam Mode

Welcome to your Medical-Surgical Nursing Exam 6! This exam is carefully designed to provide you with a realistic test-taking experience, preparing you for the pressures of an actual nursing exam.

 

Exam Details

  • Number of Questions: 50 items
  • Mode: Exam Mode

Exam Instructions

  1. Exam Mode: This mode is intended to simulate the environment of an actual exam. Questions and choices will be presented one at a time.
  2. Time Limit: Each question must be answered within 90 seconds. The entire exam should be completed within 75 minutes.
  3. Feedback and Grading: Upon completion of the exam, you will be able to see your grade and the correct answers to all questions. This will allow you to evaluate your performance and understand areas for improvement.

Tips For Success

  • Read each question carefully. You have 90 seconds per question, so make sure you understand the question before selecting your answer.
  • Pace yourself. Remember, you have 75 minutes in total, so try to maintain a steady rhythm.
  • Focus on one question at a time. Try not to worry about the questions to come.
  • Stay calm under pressure. Use your knowledge and trust your instincts.
  • Remember, it's not just about the score, but about the learning process.

This exam is not only a measurement of your current understanding, but also a valuable learning tool to prepare you for your future nursing career. Click 'Start Exam' when you're ready to begin. Good luck!

1 / 50

1. If Ms. Jones informs the nurse that her feet are cold, which nursing action would be most appropriate?

2 / 50

2. What symptom would suggest to the nurse that Ms. Taylor has suddenly developed a stationary thrombus?

3 / 50

3. A nursing assistant, tasked with giving Ms. Hanna, a patient who has undergone an abdominal hysterectomy, a sitz bath, questions why a regular bathtub can't be used. The most accurate response is based on the fact that a regular bathtub:

4 / 50

4. Why does a patient with cholelithiasis typically experience discomfort following the consumption of fatty foods?

5 / 50

5. What solution is typically used for continuous or intermittent bladder and catheter irrigations, which the nurse is assigned to monitor in a patient?

6 / 50

6. When a patient diagnosed with myasthenia gravis inquires about the cause of the disease, how should the nurse explain it, given the understanding that the condition is due to:

7 / 50

7. A nurse is educating young women about health-seeking behaviors, including the importance of the Pap test. What is the primary purpose of this test?

8 / 50

8. In the context of a legal framework, under which circumstance is a patient scheduled for a below-the-knee amputation of the right leg not allowed to sign the operative consent form?

9 / 50

9. When evaluating a patient with cholecystitis for the onset of obstructive jaundice, what symptoms would the nurse expect to observe?

10 / 50

10. While assessing for hypocalcemia, the nurse inflates a blood pressure cuff on the patient's arm, causing carpopedal spasm after approximately 3 minutes. This finding is documented as a positive:

11 / 50

11. After ear surgery, which early symptom might suggest potential damage to the motor branch of the facial nerve?

12 / 50

12. In which age group is the prevalence of chronic illness typically the highest?

13 / 50

13. What immediate post-procedure action should a nurse take after a patient has undergone a bone marrow aspiration?

14 / 50

14. A week after being diagnosed with acute renal failure, a patient enters the diuretic phase. During this stage, what condition should the patient be meticulously evaluated for?

15 / 50

15. While the exact cause of scleroderma remains unknown, what is the prevailing hypothesis regarding its origin?

16 / 50

16. When managing continuous bladder irrigation after prostate surgery, how should the flow rate be adjusted?

17 / 50

17. After a bilateral lumbar sympathectomy, a patient experiences a sudden decrease in blood pressure without any signs of bleeding. What would the nurse identify as the most probable cause?

18 / 50

18. Following a total abdominal hysterectomy, Ms. Walker experiences a slight increase in temperature and swelling in her right calf. The physician has recommended warm moist compresses for the affected leg. What is the proper nursing action when applying the warm moist compress? The nurse should:

19 / 50

19. When assigned to provide perineal care for an uncircumcised male patient, which of the following procedures is correct?

20 / 50

20. A female nurse is tasked with obtaining a medical history from a male patient suffering from urinary tract issues and sexual dysfunction. Which statement could potentially make the patient feel more comfortable and willing to discuss his condition?

21 / 50

21. If, during peritoneal dialysis, the nurse notices that the dialysate drainage from the peritoneal cavity has stopped prematurely, what should the nurse assist the patient in doing?

22 / 50

22. A nurse is tasked with teaching a patient about testicular self-examination. As part of the instructions, the nurse should mention that the most appropriate time to perform this examination is:

23 / 50

23. Following a prostatectomy, a patient experiences distressing bladder spasms. What action should the nurse take to alleviate these spasms?

24 / 50

24. What specific symptom should a nurse look for when assessing a patient with psoriasis?

25 / 50

25. For a patient with full-thickness chest burns who has received a skin graft, what occurrence at the donor site within the first 24 hours should be promptly reported?

26 / 50

26. When performing perineal care for Ms. Paris, in which direction should the nurse gently move the cotton ball or washcloth?

27 / 50

27. A patient is slated for a prostate ultrasound examination. To accurately describe the procedure to the patient, the nurse should explain that:

28 / 50

28. In patients who have had their fractured mandibles immobilized with wires, what potentially fatal complication can arise postoperatively?

29 / 50

29. What should the nurse primarily look for when assessing a patient diagnosed with bladder cancer?

30 / 50

30. What is the primary symptom reported by a patient suffering from Vincent's Angina?

31 / 50

31. Mr. Ernest underwent a vasectomy and is wondering why he needs to continue using a birth control method. What would be the most accurate response from the nurse?

32 / 50

32. While educating a group of young women about ovarian cancer, what would be an accurate statement for the nurse to make?

33 / 50

33. Ms. Lora Warren, 27 years old, has been diagnosed with a skin condition and has set an appointment at a clinic that specializes in diagnosing psoriasis. When examining Ms. Lynch's skin for psoriasis, what should the nurse be looking for?

34 / 50

34. A doctor requests that a nurse place a patient in the appropriate position for a vaginal examination. What is the typical position used for this kind of examination?

35 / 50

35. What should the nurse inform a patient with psoriasis regarding the common course of treatment?

36 / 50

36. While conducting a health behavior class for young men, the nurse discusses potential causes of penile cancer. Which of the following can be identified as a probable cause?

37 / 50

37. Ms. Anika has undergone electrocauterization of the cervix due to chronic cervicitis. What instructions should the nurse provide her after the procedure?

38 / 50

38. When a nurse is tasked with administering a conventional transcutaneous electrical nerve stimulation (TENS) device to a patient suffering from an inflamed sciatic nerve, which of the following procedures should be followed?

39 / 50

39. A nursing assistant at a conference learns that one of her patients has been diagnosed with a hydrocele. When she inquires about the standard treatment for this condition, what would be the most appropriate response?

40 / 50

40. A nurse is getting a patient ready for a pelvic examination. What instructions would the nurse likely include?

41 / 50

41. When cleaning the inner cannula of a tracheostomy tube, which solution would be the most appropriate for the nurse to use?

42 / 50

42. The nurse is preparing to administer a vaginal suppository to Ms. Davis using a specialized applicator provided with the medication. Which of the following steps is accurate?

43 / 50

43. What complication might a patient experience as a consequence of fractured ribs?

44 / 50

44. A patient scheduled for a cystectomy is curious about what the physician will be able to visualize during the procedure. What would be the most accurate response from the nurse?

45 / 50

45. Why should a nurse monitor a patient for respiratory complications after an abdominal cholecystectomy?

46 / 50

46. What is the primary complication that a nurse must assist a patient in managing when they have been diagnosed with pemphigus vulgaris?

47 / 50

47. If Ms. Potter is experiencing Candida albicans-induced vaginitis, which of the following would be a typical sign or symptom?

48 / 50

48. In order to appropriately educate men about the significance of testicular self-examination, it's crucial for the nurse to understand that testicular cancer:

49 / 50

49. What is the appropriate method for a nurse to obtain a urine sample for ketone testing from a patient's indwelling urinary catheter?

50 / 50

50. Upon noticing that a patient's knee is swollen and causing discomfort, what nursing measure should be implemented?

Text Mode

Text Mode – Text version of the exam

Questions

1. When a nurse is tasked with administering a conventional transcutaneous electrical nerve stimulation (TENS) device to a patient suffering from an inflamed sciatic nerve, which of the following procedures should be followed?

A. Place the color-coded electrodes at any location that the patient finds comfortable.
B. Modify the TENS unit’s settings until the patient experiences a sense of relief and comfort.
C. Use the TENS unit continuously throughout the day without breaks.
D. Keep the dial setting of the device consistent day after day.

2. What is the primary complication that a nurse must assist a patient in managing when they have been diagnosed with pemphigus vulgaris?

A. Dermatological lesions.
B. Disrupted digestive processes.
C. Inability to conceive.
D. Skin turning yellow.

3. When a patient diagnosed with myasthenia gravis inquires about the cause of the disease, how should the nurse explain it, given the understanding that the condition is due to:

A. A reduction in the number of operational acetylcholine receptor sites.
B. The suppression of the enzyme ACHE, which results in folded end plates.
C. A diminished quantity of the neurotransmitter acetylcholine.
D. An overproduction of the neurotransmitter dopamine.

4. What should the nurse inform a patient with psoriasis regarding the common course of treatment?

A. The removal of dead plaques through debridement.
B. Baths utilizing potassium permanganate.
C. The use of topically applied steroids.
D. Regular intake of antihistamine medications.

5. While the exact cause of scleroderma remains unknown, what is the prevailing hypothesis regarding its origin?

A. Malformation of the sebaceous glands.
B. Overproduction of melanin in the skin.
C. Problems related to eye movement.
D. An autoimmune response.

6. What is the appropriate method for a nurse to obtain a urine sample for ketone testing from a patient’s indwelling urinary catheter?

A. Employing a sterile syringe to extract the sample from a cleaned, clamped catheter.
B. Requesting the patient to urinate into a sterile container, bypassing the catheter.
C. Sterilizing the catheter with alcohol and then draining it into a sterile test tube.
D. Sanitizing the drainage valve and taking the sample from the catheter bag.

7. Why should a nurse monitor a patient for respiratory complications after an abdominal cholecystectomy?

A. The patient’s resistance is diminished due to the presence of bile in the bloodstream.
B. The patient’s fluid intake and output are imbalanced during the surgery.
C. The surgical incision is located near the diaphragm.
D. Microorganisms from the biliary tract infiltrate the bloodstream.

8. When evaluating a patient with cholecystitis for the onset of obstructive jaundice, what symptoms would the nurse expect to observe?

A. Urine of a dark hue, stools with a clay-like color, and pruritic skin.
B. Urine that is straw-colored, stools resembling putty in color, and yellowing of the sclera.
C. Poor absorption of fat-soluble vitamin K.
D. Dark-colored urine, stools that are bright green, and blotchy skin.

9. Why does a patient with cholelithiasis typically experience discomfort following the consumption of fatty foods?

A. The liver is not producing a sufficient amount of bile.
B. The gallstones are directly irritating the intestinal lining.
C. The flow of bile into the intestine is blocked.
D. Fatty foods are inherently difficult to digest.

10. What specific symptom should a nurse look for when assessing a patient with psoriasis?

A. Reddish macules on the skin.
B. Itchy skin lesions.
C. Lesions that are shiny and scaly.
D. Skin that bruises easily and heals slowly.

11. In patients who have had their fractured mandibles immobilized with wires, what potentially fatal complication can arise postoperatively?

A. Episodes of vomiting.
B. Pneumothorax or collapsed lung.
C. Hemorrhage or excessive bleeding.
D. Development of an infection.

12. What complication might a patient experience as a consequence of fractured ribs?

A. Diaphragmatic hernia.
B. Hypertension or high blood pressure.
C. Obstructive pulmonary disease.
D. Paradoxical or abnormal breathing patterns.

13. What immediate post-procedure action should a nurse take after a patient has undergone a bone marrow aspiration?

A. Administer a dose of pain medication.
B. Sterilize the site with an antiseptic solution.
C. Have the patient lie on the side that was affected.
D. Apply brief but firm pressure over the site of aspiration.

14. After a bilateral lumbar sympathectomy, a patient experiences a sudden decrease in blood pressure without any signs of bleeding. What would the nurse identify as the most probable cause?

A. Redistribution of the blood supply.
B. Elevated levels of epinephrine.
C. Side effect of postoperative pain medication.
D. Insufficient fluid intake.

15. In which age group is the prevalence of chronic illness typically the highest?

A. Infants and young children.
B. Adults of middle age.
C. Teenagers and adolescents.
D. The elderly or older adults.

16. What is the primary symptom reported by a patient suffering from Vincent’s Angina?

A. Oral ulcers that bleed.
B. Chronic fatigue and lethargy.
C. Difficulty breathing or shortness of breath.
D. Pain in the chest area.

17. If, during peritoneal dialysis, the nurse notices that the dialysate drainage from the peritoneal cavity has stopped prematurely, what should the nurse assist the patient in doing?

A. Taking deep breaths and coughing.
B. Rotating the catheter at regular intervals.
C. Shifting positions from one side to the other.
D. Lying flat on the back without a pillow.

18. After ear surgery, which early symptom might suggest potential damage to the motor branch of the facial nerve?

A. Lips and mouth feeling dry.
B. Difficulty in wrinkling the forehead.
C. Experiencing a metallic, bitter taste.
D. Sensing pain behind the ear.

19. For a patient with full-thickness chest burns who has received a skin graft, what occurrence at the donor site within the first 24 hours should be promptly reported?

A. Mild redness around the edges of the donor site.
B. Edges of the non-adherent dressing coming apart.
C. A moderate amount of serosanguinous (mixed blood and serum) discharge.
D. A small amount of oozing with a yellow-green hue.

20. A week after being diagnosed with acute renal failure, a patient enters the diuretic phase. During this stage, what condition should the patient be meticulously evaluated for?

A. Increased blood urea nitrogen (BUN) levels.
B. Metabolic acidosis.
C. High levels of potassium in the blood, or hyperkalemia.
D. A decrease in circulating blood volume, or hypovolemia.

21. While assessing for hypocalcemia, the nurse inflates a blood pressure cuff on the patient’s arm, causing carpopedal spasm after approximately 3 minutes. This finding is documented as a positive:

A. Chvostek’s sign.
B. Trousseau’s sign.
C. Romberg sign.
D. Babinski’s sign.

22. Following a prostatectomy, a patient experiences distressing bladder spasms. What action should the nurse take to alleviate these spasms?

A. Advise the patient against tensing his muscles as though he were urinating.
B. Flush the Foley catheter with 60 ml of normal saline.
C. Provide narcotic medication every 4 hours.
D. Push the catheter forward to relieve pressure on the prostatic fossa.

23. What solution is typically used for continuous or intermittent bladder and catheter irrigations, which the nurse is assigned to monitor in a patient?

A. Sterile normal saline.
B. Sodium bicarbonate solution.
C. Bacteriostatic water.
D. Plain water.

24. When managing continuous bladder irrigation after prostate surgery, how should the flow rate be adjusted?

A. Based on the patient’s oral fluid intake.
B. To maintain the drainage color as light pink.
C. To match the patient’s blood pressure readings.
D. To ensure an output of 500 ml every 8 hours.

25. While conducting a health behavior class for young men, the nurse discusses potential causes of penile cancer. Which of the following can be identified as a probable cause?

A. Undergoing circumcision.
B. Consuming a diet rich in acidic foods.
C. High consumption of caffeine.
D. Neglecting personal hygiene.

26. In the context of a legal framework, under which circumstance is a patient scheduled for a below-the-knee amputation of the right leg not allowed to sign the operative consent form?

A. If the patient has recently been given any sedative medication.
B. If a comprehensive medical history and physical examination have not been completed.
C. If a discussion about alternative treatments with two physicians hasn’t been conducted and documented.
D. If the patient has not been given enough time to consider the decision.

27. A female nurse is tasked with obtaining a medical history from a male patient suffering from urinary tract issues and sexual dysfunction. Which statement could potentially make the patient feel more comfortable and willing to discuss his condition?

A. “Do you perceive a connection between your sexual dysfunction and your urinary tract issue?”
B. “Can you recall when you first started experiencing this issue?”
C. “What makes you believe you have a problem?”
D. “Do you feel embarrassed discussing this issue with a female nurse?”

28. When assigned to provide perineal care for an uncircumcised male patient, which of the following procedures is correct?

A. Retract the foreskin and cleanse the area beneath it.
B. Cleanse the anal area separately, at a different time.
C. Thoroughly wash the scrotum using sterile normal saline.
D. Wash the entire perineal area with an antiseptic solution.

29. A patient is slated for a prostate ultrasound examination. To accurately describe the procedure to the patient, the nurse should explain that:

A. The procedure involves the use of x-rays to generate a visual image.
B. The procedure involves a large amount of radiation exposure.
C. The procedure is conducted using a cystoscope.
D. A probe will be introduced into the rectum.

30. A nurse is tasked with teaching a patient about testicular self-examination. As part of the instructions, the nurse should mention that the most appropriate time to perform this examination is:

A. Following a warm bath or shower.
B. After physical exercise.
C. Right after rising from bed in the morning.
D. Just before going to bed at night.

31. Mr. Ernest underwent a vasectomy and is wondering why he needs to continue using a birth control method. What would be the most accurate response from the nurse?

A. A certain amount of live sperm may remain in the ejaculatory fluid for a while.
B. Despite having undergone a vasectomy, it’s still recommended to use a condom for 1 to 2 years.
C. The sperm count will not register as negative until his testosterone level decreases.
D. Vasectomy doesn’t impact fertility immediately; it takes a few weeks for the body to adjust.

32. In order to appropriately educate men about the significance of testicular self-examination, it’s crucial for the nurse to understand that testicular cancer:

A. Is more common in older men above the age of 60.
B. Has a high incidence of metastasis at early stages.
C. Is not identifiable through laboratory tests.
D. Seldom metastasizes.

33. A patient scheduled for a cystectomy is curious about what the physician will be able to visualize during the procedure. What would be the most accurate response from the nurse?

A. The bladder and the kidneys.
B. The kidneys and the ureters.
C. The urethra and the bladder.
D. The prostate and the ureters.

34. What should the nurse primarily look for when assessing a patient diagnosed with bladder cancer?

A. Pain above the pubic bone.
B. Painful urination.
C. Blood in the urine without any pain.
D. Inability to completely empty the bladder.

35. A nursing assistant at a conference learns that one of her patients has been diagnosed with a hydrocele. When she inquires about the standard treatment for this condition, what would be the most appropriate response?

A. The problem may necessitate surgical intervention.
B. The issue typically requires extensive medical or surgical intervention.
C. Medication is usually effective in controlling fluid accumulation.
D. Regular exercise and a healthy diet typically resolve the issue.

36. While educating a group of young women about ovarian cancer, what would be an accurate statement for the nurse to make?

A. The initial symptoms of ovarian cancer tend to be unclear.
B. An irregular menstrual cycle is the most noticeable early symptom.
C. Ovarian cancer only affects women after menopause.
D. Chemotherapy isn’t employed in the treatment of ovarian cancer.

37. If Ms. Potter is experiencing Candida albicans-induced vaginitis, which of the following would be a typical sign or symptom?

A. Sudden weight gain.
B. Upper abdominal discomfort.
C. Reduced urinary output.
D. Severe itching in the vaginal and perineal areas.

38. When performing perineal care for Ms. Paris, in which direction should the nurse gently move the cotton ball or washcloth?

A. Ascending from the anal region towards the pubic area.
B. Zigzagging from the left to the right across the labia majora.
C. In a side-to-side motion across the labia majora.
D. Descending from the pubic area towards the anus.

39. A nurse is getting a patient ready for a pelvic examination. What instructions would the nurse likely include?

A. Request that the client urinates prior to the examination.
B. Instruct the client to maintain shallow breathing throughout the examination.
C. Inform the client that anesthesia will be administered for the examination.
D. Assure the client that the examination will cause no discomfort.

40. A nurse is educating young women about health-seeking behaviors, including the importance of the Pap test. What is the primary purpose of this test?

A. To identify cervical cancer.
B. To diagnose ovarian cysts.
C. To detect infections in the uterus.
D. To confirm early pregnancy.

41. A doctor requests that a nurse place a patient in the appropriate position for a vaginal examination. What is the typical position used for this kind of examination?

A. Patient lies on their back with legs bent and feet flat (Dorsal recumbent position).
B. Patient lies on their left side with right knee bent and raised towards the chest (Sim’s position).
C. Patient lies on their back with legs bent, hips rotated outward, and feet in stirrups (Lithotomy position).
D. Patient lies on their stomach with arms at the side (Prone position).

42. Ms. Anika has undergone electrocauterization of the cervix due to chronic cervicitis. What instructions should the nurse provide her after the procedure?

A. Refrain from straining and lifting heavy objects until approved by the physician.
B. Rest in bed for the following five days.
C. Use a douche the next day to clear out debris and blood clots.
D. Consume a diet high in spicy foods to promote healing.

43. A nursing assistant, tasked with giving Ms. Hanna, a patient who has undergone an abdominal hysterectomy, a sitz bath, questions why a regular bathtub can’t be used. The most accurate response is based on the fact that a regular bathtub:

A. Directs heat to the legs, altering the intended effect of heat applied to the pelvic region.
B. Is unable to provide water at the optimal temperature required for this procedure.
C. May cause an allergic reaction due to the commonly used cleaning products.
D. Does not maintain the same level of cleanliness as a specialized sitz bath tub.

44. When cleaning the inner cannula of a tracheostomy tube, which solution would be the most appropriate for the nurse to use?

A. A solution of povidone-iodine.
B. A solution of baking soda.
C. Hydrogen peroxide.
D. Sodium hydrochloride.

45. Upon noticing that a patient’s knee is swollen and causing discomfort, what nursing measure should be implemented?

A. Assist the patient in changing positions to alleviate discomfort.
B. Promote exercises that engage the quadriceps muscles.
C. Encourage the patient to bend the knee repeatedly.
D. Conduct passive range of motion exercises during each shift.

46. The nurse is preparing to administer a vaginal suppository to Ms. Davis using a specialized applicator provided with the medication. Which of the following steps is accurate?

A. Apply petroleum jelly to the tip of the suppository.
B. Insert the applicator about ½ inch deep and then press the plunger.
C. Gently insert the applicator tip in an upward and forward motion.
D. Apply alcohol to the suppository before insertion.

47. What symptom would suggest to the nurse that Ms. Taylor has suddenly developed a stationary thrombus?

A. Knee swelling.
B. Severe headache.
C. Frequent hiccups.
D. Foot numbness.

48. Following a total abdominal hysterectomy, Ms. Walker experiences a slight increase in temperature and swelling in her right calf. The physician has recommended warm moist compresses for the affected leg. What is the proper nursing action when applying the warm moist compress? The nurse should:

A. Wrap the moist gauze in a towel.
B. Maintain a sterile technique.
C. Heat the water to 120°F.
D. Keep the compress on for a continuous 24 hours.

49. If Ms. Jones informs the nurse that her feet are cold, which nursing action would be most appropriate?

A. Use an electric heating pad.
B. Place her feet in a basin of cold water.
C. Elevate her feet on a stool.
D. Envelop them in a warm blanket.

50. Ms. Lora Warren, 27 years old, has been diagnosed with a skin condition and has set an appointment at a clinic that specializes in diagnosing psoriasis. When examining Ms. Lynch’s skin for psoriasis, what should the nurse be looking for?

A. Red patches covered with silvery scales.
B. Oozing lesions on the body’s trunk.
C. A rash marked by raised, pus-filled lesions.
D. Dark, rough patches on the hands and feet.

Answers and Rationales

1. Correct answer:

B. Modify the TENS unit’s settings until the patient experiences a sense of relief and comfort. In using a TENS device, the settings should be adjusted according to the patient’s feedback. This includes the pulse rate, pulse width, and intensity. The goal is to achieve a sensation that is strong but comfortable and provides the greatest pain relief.

Incorrect answer options:

A. Place the color-coded electrodes at any location that the patient finds comfortable. This is incorrect because electrode placement in TENS therapy should be guided by the location of pain or by specific nerve distribution, not just based on patient comfort. Proper placement is crucial to ensure the electrical impulses target the correct area and provide optimal pain relief.

C. Use the TENS unit continuously throughout the day without breaks. Continuous use of the TENS unit throughout the day without breaks is not typically recommended. While TENS can be used frequently, there should be periods of rest between sessions. Overuse can lead to skin irritation at the site of electrode placement and may also decrease the effectiveness of the treatment over time.

D. Keep the dial setting of the device consistent day after day. Keeping the dial setting of the device consistent day after day is not the best approach, as the effectiveness of TENS can decrease with continuous use at the same settings (a phenomenon known as “accommodation”). It’s often recommended to adjust the settings, either by changing the pulse width, pulse rate, or intensity, or by using a different mode of stimulation if available.

2. Correct answer:

A. Dermatological lesions. Pemphigus vulgaris is an autoimmune skin disorder characterized by the formation of blisters and sores (lesions) on the skin and mucous membranes. These lesions can cause discomfort, pain, and potential secondary infection if not properly managed. Therefore, a primary nursing responsibility is to assist with wound care, pain management, and the prevention of infections.

Incorrect answer options:

B. Disrupted digestive processes. While pemphigus vulgaris can cause painful sores in the mouth and throat that may make it difficult for the patient to eat and swallow, it does not directly disrupt digestive processes. However, maintaining adequate nutrition is an important part of managing the condition, and a nurse may assist with strategies to make eating more comfortable.

C. Inability to conceive. Pemphigus vulgaris does not directly impact fertility or the ability to conceive. It is a skin condition and does not affect the reproductive system.

D. Skin turning yellow. Pemphigus vulgaris does not cause the skin to turn yellow. Yellowing of the skin is typically associated with conditions that affect the liver or bile production, such as jaundice.

3. Correct answer:

A. A reduction in the number of operational acetylcholine receptor sites. Myasthenia gravis is an autoimmune disease where the body’s immune system produces antibodies that block or destroy the acetylcholine receptors at the junction between the nerves and muscles. This reduces the number of operational receptor sites and impairs the communication between nerves and muscles, resulting in muscle weakness.

Incorrect answer options:

B. The suppression of the enzyme ACHE, which results in folded end plates. This statement is incorrect. Myasthenia gravis is not caused by the suppression of acetylcholinesterase (ACHE), the enzyme that breaks down acetylcholine in the synaptic cleft. In fact, medications that inhibit ACHE are sometimes used to treat myasthenia gravis by increasing the amount of acetylcholine available to stimulate the receptors.

C. A diminished quantity of the neurotransmitter acetylcholine. This statement is incorrect. In myasthenia gravis, the quantity of acetylcholine is not diminished. Rather, the number of receptors for acetylcholine on the muscle cells is reduced due to attack by the body’s own immune system.

D. An overproduction of the neurotransmitter dopamine. This statement is incorrect. Myasthenia gravis is not caused by an overproduction of dopamine. Dopamine is a neurotransmitter that plays a role in many different neurological functions, but it is not directly involved in the pathophysiology of myasthenia gravis.

4. Correct answer:

C. The use of topically applied steroids. Topical steroids are frequently used as a first-line treatment for psoriasis. They reduce inflammation and help to slow the rapid growth of skin cells that leads to psoriasis plaques. They can be particularly effective for mild to moderate cases of psoriasis.

Incorrect answer options:

A. The removal of dead plaques through debridement. While it’s true that managing psoriasis often involves removing scales or plaques, debridement (the medical removal of dead, damaged, or infected tissue) is not typically used. Instead, treatments like moisturizers, salicylic acid, or other scale-softening agents are often recommended.

B. Baths utilizing potassium permanganate. Potassium permanganate baths are not commonly used in the treatment of psoriasis. While bathing can help to soften psoriasis plaques and soothe the skin, it’s usually recommended to use a mild soap and to moisturize the skin immediately after bathing.

D. Regular intake of antihistamine medications. Antihistamines are not typically used to treat psoriasis. While they can help with itching, they do not address the underlying issue of skin cell overproduction and inflammation that characterizes psoriasis.

5. Correct answer:

D. An autoimmune response. Scleroderma is believed to be an autoimmune condition. In autoimmune diseases, the body’s immune system mistakenly attacks its own tissues. In the case of scleroderma, the disease results in the overproduction of collagen, leading to the hardening and tightening of the skin and connective tissues. It can also affect internal organs and blood vessels.

Incorrect answer options:

A. Malformation of the sebaceous glands. This statement is incorrect. Scleroderma is not caused by a malformation of the sebaceous glands. Sebaceous glands produce sebum, an oily substance that helps to moisturize the skin. Their dysfunction is associated with conditions like acne, but not scleroderma.

B. Overproduction of melanin in the skin. This statement is incorrect. Scleroderma is not caused by an overproduction of melanin in the skin. Melanin is the pigment responsible for skin color, and conditions related to its overproduction typically involve skin discoloration or darkening, not hardening or tightening as seen in scleroderma.

C. Problems related to eye movement. This statement is incorrect. Scleroderma does not originate from problems related to eye movement. While it can affect many parts of the body, it is primarily a condition that affects the skin and connective tissues.

6. Correct answer:

A. Employing a sterile syringe to extract the sample from a cleaned, clamped catheter. The appropriate method to collect a urine sample from an indwelling catheter for testing is by using a sterile syringe to withdraw the urine from the catheter after cleaning the access port and momentarily clamping the catheter. This method ensures that fresh urine is obtained for testing and minimizes the risk of contamination from the catheter or the drainage bag.

Incorrect answer options:

B. Requesting the patient to urinate into a sterile container, bypassing the catheter. This option is incorrect because a patient with an indwelling urinary catheter cannot urinate independently into a container. The catheter is designed to continuously drain the bladder into a collection bag.

C. Sterilizing the catheter with alcohol and then draining it into a sterile test tube. This option is incorrect. Sterilizing the catheter with alcohol is not a recommended practice as it may damage the catheter and harm the patient’s urethral tissue. Moreover, draining the catheter directly into a test tube may increase the risk of contamination.

D. Sanitizing the drainage valve and taking the sample from the catheter bag. This option is incorrect. Urine samples for laboratory testing should not be taken from the drainage bag because the urine in the bag may have been there for some time and may be contaminated. It’s always best to obtain a fresh urine sample from the catheter itself using a sterile syringe.

7. Correct answer:

C. The surgical incision is located near the diaphragm. An abdominal cholecystectomy involves the removal of the gallbladder through an incision in the abdomen. Because the gallbladder is located near the diaphragm, surgery and post-surgical pain can affect diaphragmatic movement, making deep breathing and coughing more difficult for the patient. This can lead to atelectasis (collapse of part or all of a lung) and pneumonia, which is why nurses monitor patients for respiratory complications after such a surgery.

Incorrect answer options:

A. The patient’s resistance is diminished due to the presence of bile in the bloodstream. This statement is incorrect. While bile leakage can occur during a cholecystectomy, it doesn’t commonly lead to decreased resistance and subsequent respiratory complications. In fact, the body’s immune system typically responds to such an occurrence by increasing resistance.

B. The patient’s fluid intake and output are imbalanced during the surgery. While maintaining fluid balance is critical during any surgery, it’s not the primary reason for monitoring respiratory function following an abdominal cholecystectomy. Fluid imbalances can lead to complications such as electrolyte imbalance, kidney problems, and heart failure, but they do not directly result in respiratory complications.

D. Microorganisms from the biliary tract infiltrate the bloodstream. While there is a risk of infection during any surgery, this isn’t the primary reason for monitoring respiratory function following an abdominal cholecystectomy. Any infection that does occur would likely be due to surgical contamination and would be systemic, not specifically respiratory.

8. Correct answer:

A. Urine of a dark hue, stools with a clay-like color, and pruritic skin. Obstructive jaundice occurs when there is a blockage in the bile duct system, such as with gallstones in cholecystitis, which prevents bilirubin from being excreted from the body. This leads to an accumulation of bilirubin in the bloodstream and tissues, resulting in symptoms such as dark urine (due to bilirubin being excreted through the kidneys), clay-colored or pale stools (due to a lack of bilirubin reaching the intestines), and pruritic (itchy) skin (due to the accumulation of bile salts under the skin).

Incorrect answer options:

B. Urine that is straw-colored, stools resembling putty in color, and yellowing of the sclera. This option is incorrect because straw-colored urine is usually normal. Jaundice typically causes urine to be dark or brown-colored. Putty-colored stools could be a sign of jaundice but it’s more common to describe them as clay-colored. Yellowing of the sclera is a symptom of jaundice but it’s not the only symptom to look out for.

C. Poor absorption of fat-soluble vitamin K. This statement is incorrect. While poor absorption of fat-soluble vitamins, including vitamin K, can occur in conditions that affect the liver and biliary system, it’s not a primary symptom of obstructive jaundice. This condition may cause bleeding problems due to vitamin K deficiency, but these are not the first signs of the condition.

D. Dark-colored urine, stools that are bright green, and blotchy skin. This statement is incorrect. Although dark urine can be a sign of obstructive jaundice, bright green stools and blotchy skin are not typical symptoms. Green stools might indicate rapid transit through the intestines, and blotchy skin can have many causes, but these are not typically associated with obstructive jaundice.

9. Correct answer:

C. The flow of bile into the intestine is blocked. A patient with cholelithiasis, or gallstones, typically experiences discomfort after consuming fatty foods because the gallstones can block the flow of bile from the gallbladder into the intestine. Bile is necessary for the digestion and absorption of fats. When a person consumes fatty foods, the gallbladder contracts to release bile into the small intestine. If gallstones are blocking the bile duct, the gallbladder’s contraction can cause pain and discomfort, commonly known as biliary colic.

Incorrect answer options:

A. The liver is not producing a sufficient amount of bile. This statement is incorrect. In cholelithiasis, the problem is not with the production of bile by the liver but rather with the flow of bile from the gallbladder due to the presence of gallstones.

B. The gallstones are directly irritating the intestinal lining. This statement is incorrect. Gallstones typically cause discomfort by obstructing the flow of bile, not by directly irritating the intestinal lining.

D. Fatty foods are inherently difficult to digest. This statement is not entirely correct. Fatty foods can be more challenging to digest, but they are not inherently difficult to digest for a healthy individual with a properly functioning gallbladder and digestive system. In the context of cholelithiasis, the issue is the obstruction of bile flow, which makes the digestion of fats more difficult and can cause discomfort.

10. Correct answer:

C. Lesions that are shiny and scaly. Psoriasis is a chronic autoimmune skin disorder characterized by the presence of raised, reddish, often scaly patches of skin, known as plaques. These plaques are typically covered with silvery scales and can occur anywhere on the body, but they are most commonly found on the elbows, knees, scalp, and lower back.

Incorrect answer options:

A. Reddish macules on the skin. Macules are flat, discolored spots on the skin, and they are not typically associated with psoriasis. Instead, psoriasis is characterized by raised, red, scaly patches known as plaques.

B. Itchy skin lesions. While itching can be a symptom of psoriasis, it does not specifically define the condition. Many skin conditions can cause itching, so this symptom alone is not sufficient to diagnose psoriasis.

D. Skin that bruises easily and heals slowly. Easy bruising and slow healing are not typical symptoms of psoriasis. These symptoms may be associated with other medical conditions, such as certain blood disorders or diabetes, but they are not characteristic of psoriasis.

11. Correct answer:

A. Episodes of vomiting. In patients with mandibular fractures who have had their jaws wired shut, episodes of vomiting can be potentially fatal. This is because the vomit cannot be expelled properly through the mouth due to the immobilization of the jaw. As a result, there is a risk of aspiration, where the vomit is inhaled into the lungs. Aspiration can lead to serious complications like aspiration pneumonia, a severe and potentially fatal lung infection.

Incorrect answer options:

B. Pneumothorax or collapsed lung. Pneumothorax, or collapsed lung, is not a typical postoperative complication of mandibular fracture immobilization with wires. While it’s a serious condition, it’s usually related to trauma or procedures involving the chest and lungs.

C. Hemorrhage or excessive bleeding. While hemorrhage or excessive bleeding is a potential complication of any surgical procedure, it is not uniquely associated with mandibular fracture repair and immobilization. Furthermore, it’s not typically a fatal complication if managed appropriately.

D. Development of an infection. An infection is also possible after any procedure, but is not immediately fatal in itself like aspiration of vomit can potentially be.

12. Correct answer:

D. Paradoxical or abnormal breathing patterns. A fractured rib can result in a condition known as flail chest if multiple adjacent ribs are broken in multiple places. This results in a segment of the rib cage that is not attached to the rest of the chest wall structure. During breathing, this detached segment moves in the opposite direction to the rest of the chest wall, creating a paradoxical or abnormal breathing pattern. This is a serious condition that can impair oxygenation and ventilation, leading to respiratory distress.

Incorrect answer options:

A. Diaphragmatic hernia. While it’s theoretically possible for a rib fracture to lead to a diaphragmatic hernia if the fractured rib were to penetrate the diaphragm, this would be an extremely rare and unusual event. Diaphragmatic hernias are more commonly associated with traumatic injury to the abdomen or congenital defects.

B. Hypertension or high blood pressure. Hypertension is not typically a direct consequence of fractured ribs. While pain from the injury could potentially cause a temporary increase in blood pressure, it’s not a long-term complication related to rib fractures.

C. Obstructive pulmonary disease. Obstructive pulmonary disease, which includes conditions like chronic obstructive pulmonary disease (COPD) and asthma, is characterized by long-term difficulty in getting air out of the lungs. This is not typically a consequence of rib fractures.

13. Correct answer:

D. Apply brief but firm pressure over the site of aspiration. After a bone marrow aspiration, the nurse should apply brief but firm pressure over the site of aspiration to prevent bleeding. Once the bleeding has stopped, a sterile dressing should be applied to the site.

Incorrect answer options:

A. Administer a dose of pain medication. While pain medication may be needed after a bone marrow aspiration, it is not the immediate action a nurse should take. The immediate concern is to prevent bleeding at the site of aspiration.

B. Sterilize the site with an antiseptic solution. Sterilizing the site with an antiseptic solution is done before the procedure, not after. After the procedure, the primary concern is to prevent bleeding.

C. Have the patient lie on the side that was affected. Having the patient lie on the side that was affected is not typically necessary after a bone marrow aspiration. The immediate concern is to prevent bleeding at the site of aspiration.

14. Correct answer:

A. Redistribution of the blood supply. A bilateral lumbar sympathectomy involves the surgical removal or interruption of part of the sympathetic nervous system in the lumbar region. One of the physiological effects of this procedure is vasodilation or widening of the blood vessels, especially in the lower extremities. This vasodilation can lead to a redistribution of the blood supply, resulting in a sudden drop in blood pressure, a condition known as postoperative hypotension.

Incorrect answer options:

B. Elevated levels of epinephrine. Elevated levels of epinephrine, also known as adrenaline, would typically cause an increase, not a decrease, in blood pressure. Epinephrine is a hormone that stimulates the heart rate, constricts blood vessels, and dilates air passages, allowing increased blood flow to muscles and improved oxygen flow to the lungs.

C. Side effects of postoperative pain medication. While some pain medications can potentially cause a decrease in blood pressure, in this specific context of a bilateral lumbar sympathectomy, the sudden drop in blood pressure is more likely due to the redistribution of blood supply rather than a side effect of pain medication.

D. Insufficient fluid intake. Insufficient fluid intake can lead to dehydration and potentially a decrease in blood pressure. However, in the immediate postoperative period, patients are typically closely monitored and receive fluids intravenously, making insufficient fluid intake an unlikely cause for a sudden drop in blood pressure.

15. Correct answer:

D. The elderly or older adults. The prevalence of chronic illness is typically highest in the elderly or older adult population. As people age, they are more likely to develop chronic health conditions such as heart disease, cancer, stroke, diabetes, and chronic respiratory diseases, among others. These chronic conditions can have a significant impact on their quality of life and may require long-term management and care.

Incorrect answer options:

A. Infants and young children. While infants and young children can develop chronic illnesses, their prevalence is generally lower than in the elderly population. However, early detection and appropriate management of chronic conditions in children are crucial for their long-term health outcomes.

B. Adults of middle age. Adults of middle age may begin to experience an increase in chronic illnesses compared to younger age groups, but the prevalence is still typically lower than in the elderly population.

C. Teenagers and adolescents. Teenagers and adolescents can also develop chronic illnesses, but the prevalence is generally lower than in the elderly population. It is essential to promote healthy lifestyle choices and appropriate healthcare management during adolescence to reduce the risk of developing chronic conditions later in life.

16. Correct answer:

A. Oral ulcers that bleed. Vincent’s Angina, also known as trench mouth or acute necrotizing ulcerative gingivitis (ANUG), is a severe form of gum disease. It primarily presents with painful oral ulcers that may bleed, foul breath, and may be accompanied by fever and swollen lymph nodes.

Incorrect answer options:

B. Chronic fatigue and lethargy. While chronic fatigue and lethargy can occur with many illnesses, they are not the primary symptoms of Vincent’s Angina. The primary symptom of this condition is painful, bleeding oral ulcers.

C. Difficulty breathing or shortness of breath. Difficulty breathing or shortness of breath is more commonly associated with respiratory conditions, not Vincent’s Angina. The primary symptom of this condition is painful, bleeding oral ulcers.

D. Pain in the chest area. Chest pain is a common symptom of heart disease and other conditions affecting the chest area, not Vincent’s Angina. The primary symptom of this condition is painful, bleeding oral ulcers.

17. Correct answer:

C. Shifting positions from one side to the other. If the drainage of dialysate stops prematurely during peritoneal dialysis, it could be due to the catheter being kinked or occluded. The nurse should assist the patient in shifting positions from one side to the other, which can help dislodge any fibrin clot or omental wrapping that might be obstructing the catheter and thus restore the flow of dialysate.

Incorrect answer options:

A. Taking deep breaths and coughing. While deep breaths and coughing can be helpful in some situations to improve lung function or dislodge foreign bodies in the respiratory tract, they are unlikely to affect the drainage of dialysate during peritoneal dialysis.

B. Rotating the catheter at regular intervals. Rotating the catheter is not a recommended practice as it may cause trauma to the peritoneal lining or lead to dislocation or migration of the catheter. It is not likely to solve the problem of premature stoppage of dialysate drainage.

D. Lying flat on the back without a pillow. Lying flat on the back without a pillow does not typically facilitate the drainage of dialysate during peritoneal dialysis. Changing the patient’s position to either side is a more effective strategy.

18. Correct answer:

B. Difficulty in wrinkling the forehead. The facial nerve, or cranial nerve VII, is responsible for motor control of most of the muscles of facial expression, including those that wrinkle the forehead. Damage to this nerve during ear surgery can cause weakness or paralysis of these muscles, which could manifest as difficulty in wrinkling the forehead.

Incorrect answer options:

A. Lips and mouth feeling dry. Dryness of the lips and mouth could be due to various reasons, including dehydration or side effects of certain medications, and is not typically associated with damage to the facial nerve.

C. Experiencing a metallic, bitter taste. Altered taste sensations, especially a metallic or bitter taste, can be associated with damage to the chorda tympani, a branch of the facial nerve that carries taste sensations from the anterior two-thirds of the tongue. However, it’s not typically related to the motor functions of the facial nerve.

D. Sensing pain behind the ear. Pain behind the ear could be related to the surgical procedure itself or other complications, but it’s not a typical symptom of damage to the motor branch of the facial nerve.

19. Correct answer:

D. A small amount of oozing with a yellow-green hue. Oozing with a yellow-green hue may indicate the presence of pus, suggesting an infection at the donor site. This should be reported promptly to the healthcare provider for appropriate management to prevent further complications, such as cellulitis or systemic infection.

Incorrect answer options:

A. Mild redness around the edges of the donor site. Mild redness around the edges of the donor site is a normal inflammatory response and is usually expected within the first 24 hours after skin graft surgery. It does not typically indicate a problem that needs to be reported immediately.

B. Edges of the non-adherent dressing coming apart. The edges of the non-adherent dressing coming apart could be an issue regarding dressing maintenance and does not necessarily indicate a complication. However, it is crucial to ensure the dressing is intact to protect the site from infection.

C. A moderate amount of serosanguinous (mixed blood and serum) discharge. A moderate amount of serosanguinous discharge is generally expected after skin graft surgery as a part of the normal healing process. However, excessive or persistent serosanguinous discharge might need to be reported.

20. Correct answer:

D. A decrease in circulating blood volume, or hypovolemia. During the diuretic phase of acute renal failure, the kidneys have recovered their ability to excrete wastes, but not to concentrate the urine. This can lead to large amounts of dilute urine being produced, which can result in a significant loss of fluid and electrolytes from the body, potentially leading to hypovolemia. Hypovolemia can cause low blood pressure, rapid heart rate, and other serious complications, so it is important to monitor patients carefully during this phase.

Incorrect answer options:

A. Increased blood urea nitrogen (BUN) levels. While high BUN levels can occur in acute renal failure, they are more typically seen in the oliguric phase, when the kidneys are not properly filtering waste from the blood. During the diuretic phase, BUN levels generally start to decrease as kidney function improves.

B. Metabolic acidosis. Metabolic acidosis is another complication that can occur in the oliguric phase of acute renal failure, when the kidneys are not effectively excreting acids. In the diuretic phase, acid-base balance generally starts to improve.

C. High levels of potassium in the blood, or hyperkalemia. Hyperkalemia can occur in the oliguric phase of acute renal failure, when the kidneys are not effectively excreting potassium. However, during the diuretic phase, the risk shifts toward hypokalemia, or low potassium levels, due to the large amounts of dilute urine being excreted.

21. Correct answer:

B. Trousseau’s sign. Trousseau’s sign is a medical sign observed in patients with hypocalcemia (low serum calcium levels). When the blood pressure cuff is inflated on a patient’s arm for about 3 minutes, it can induce carpopedal spasm (a spasm of the hand, thumb, foot, or toes), which is indicative of a positive Trousseau’s sign.

Incorrect answer options:

A. Chvostek’s sign. Chvostek’s sign is another indicator of hypocalcemia, but it involves facial twitching in response to tapping the facial nerve.

C. Romberg sign. The Romberg sign is used to investigate ataxia or loss of coordination. A positive Romberg sign is when a patient sways or falls when standing upright with feet together and eyes closed.

D. Babinski’s sign. The Babinski sign is used to evaluate the function of the central nervous system, particularly the corticospinal tract. A positive Babinski sign is when the big toe extends upward and the other toes fan out upon stimulating the sole of the foot, suggesting a neurological condition.

22. Correct answer:

A. Advise the patient against tensing his muscles as though he were urinating. After a prostatectomy, bladder spasms can occur due to irritation or stimulation of the bladder muscles. Patients should be advised not to tense their muscles as if they were urinating because this can trigger spasms. Instead, they should try to relax their pelvic muscles to help alleviate the spasms.

Incorrect answer options:

B. Flush the Foley catheter with 60 ml of normal saline. Flushing a Foley catheter should only be done when it is blocked or as per physician’s order. Routine flushing is not typically recommended as it can increase the risk of urinary tract infection. This action would not directly address bladder spasms.

C. Provide narcotic medication every 4 hours. Narcotic medication can help manage severe pain, but it is not the first-line treatment for bladder spasms. Moreover, narcotics can cause side effects such as constipation, nausea, and potential addiction.

D. Push the catheter forward to relieve pressure on the prostatic fossa. Pushing the catheter forward can cause pain and potential harm. It may not alleviate the bladder spasms and could potentially cause further irritation and discomfort.

23. Correct answer:

A. Sterile normal saline. For bladder and catheter irrigations, sterile normal saline is typically used. It is isotonic and does not cause any harm to the bladder tissues. It is also effective at removing any debris or clots that might be present in the bladder.

Incorrect answer options:

B. Sodium bicarbonate solution. This solution is not typically used for bladder irrigations. Sodium bicarbonate can alter the pH of the urine and is not typically recommended for bladder or catheter irrigation.

C. Bacteriostatic water. Bacteriostatic water contains a preservative to prevent the growth of bacteria. While it is used for diluting or dissolving medications for injection, it is not typically used for bladder or catheter irrigation.

D. Plain water. Plain water is hypotonic and can cause lysis (destruction) of the cells in the bladder wall if used for irrigation. Therefore, it is not recommended for bladder or catheter irrigation.

24. Correct answer:

B. To maintain the drainage color as light pink. The flow rate of continuous bladder irrigation (CBI) after prostate surgery is typically adjusted to maintain the drainage color as light pink. This indicates that the bladder is being sufficiently irrigated to dilute blood and prevent the formation of clots.

Incorrect answer options:

A. Based on the patient’s oral fluid intake. The patient’s oral fluid intake does not directly influence the rate of CBI. The irrigation flow rate is determined by the need to clear the bladder of blood and clots, not by how much fluid the patient is consuming orally.

C. To match the patient’s blood pressure readings. The rate of CBI is not set according to blood pressure readings. Instead, it’s based on the appearance of the urinary output to ensure that it is free from clots and is of a light pink color.

D. To ensure an output of 500 ml every 8 hours. There is no specific output volume that needs to be achieved with CBI. The goal is to keep the urine light pink and free of clots, and this might require more or less than 500 ml every 8 hours, depending on the patient’s specific circumstances.

25. Correct answer:

D. Neglecting personal hygiene. Neglecting personal hygiene, specifically in the genital area, can be a risk factor for penile cancer. Poor hygiene can lead to chronic inflammation and infections that may increase the risk of developing penile cancer.

Incorrect answer options:

A. Undergoing circumcision. Contrarily, circumcision, especially if performed soon after birth, can actually reduce the risk of penile cancer. This is because the procedure helps maintain cleanliness and reduces the likelihood of infections and inflammation.

B. Consuming a diet rich in acidic foods. There’s no scientific evidence that directly links a diet rich in acidic foods to penile cancer. Although a balanced diet is crucial for overall health and disease prevention, specific food choices aren’t directly associated with penile cancer risk.

C. High consumption of caffeine. Caffeine consumption isn’t considered a risk factor for penile cancer. While excessive caffeine intake can have negative effects on overall health, it’s not directly associated with the development of penile cancer.

26. Correct answer:

A. If the patient has recently been given any sedative medication. Patients who have recently received sedative medications are not allowed to sign consent forms, as the medications may impair their judgment, understanding, and ability to make informed decisions. Informed consent requires that the patient is of sound mind and understanding of the procedure, its risks, benefits, and alternatives.

Incorrect answer options:

B. If a comprehensive medical history and physical examination have not been completed. While these are important components of preoperative care, they don’t directly affect a patient’s capacity to provide informed consent. However, the information from these assessments can help inform the discussion with the patient about the risks and benefits of the procedure.

C. If a discussion about alternative treatments with two physicians hasn’t been conducted and documented. While it’s important for a patient to be aware of all treatment options, it is not a legal requirement that the patient must discuss these with two physicians. The key is that the patient is fully informed about the procedure and all reasonable alternatives.

D. If the patient has not been given enough time to consider the decision. While it’s crucial that patients are given sufficient time to make an informed decision, there is no legal requirement specifying a set amount of time for consideration. However, rushing a patient into making a decision can undermine the process of informed consent.

27. Correct answer:

B. “Can you recall when you first started experiencing this issue?” This question is neutral, nonjudgmental, and specific. It focuses on the timeline of the patient’s symptoms, which is a crucial aspect of a medical history. It does not make assumptions about the patient’s feelings or thoughts about their condition or the conversation.

Incorrect answer options:

A. “Do you perceive a connection between your sexual dysfunction and your urinary tract issue?” This question assumes that the patient has a certain level of medical knowledge or understanding of their condition. It could potentially make the patient feel uncomfortable or inadequate if they do not understand the possible connection.

C. “What makes you believe you have a problem?” This question can come across as challenging or dismissive of the patient’s experiences or concerns, which can make them feel uncomfortable or defensive.

D. “Do you feel embarrassed discussing this issue with a female nurse?” This question directly addresses the patient’s potential discomfort due to the gender of the nurse, which can make the situation more awkward or uncomfortable. Instead, the nurse should maintain a professional demeanor and focus on the patient’s symptoms and experiences.

28. Correct answer:

A. Retract the foreskin and cleanse the area beneath it. In the provision of perineal care for an uncircumcised male patient, the correct procedure involves gently retracting the foreskin to cleanse the area beneath it. This is necessary to maintain hygiene and prevent the buildup of smegma, which can lead to infections, such as balanitis.

Incorrect answer options:

B. Cleanse the anal area separately, at a different time. While it’s essential to be thorough in cleansing all parts of the perineal area, there is no specific requirement to cleanse the anal area at a different time. During a perineal care procedure, the anal area is typically cleansed last to prevent the spread of fecal matter and bacteria to the other areas.

C. Thoroughly wash the scrotum using sterile normal saline. While sterile normal saline can be used for wound irrigation or certain types of cleansing, it’s generally not necessary for routine perineal care. Usually, warm water and mild soap are sufficient for cleansing the scrotum and other areas of the perineum.

D. Wash the entire perineal area with an antiseptic solution. Using an antiseptic solution to wash the entire perineal area is generally not recommended for routine perineal care. Antiseptics can cause irritation and dryness of the skin, and their overuse can disrupt the skin’s natural flora, potentially leading to an increased risk of infection. Warm water and mild soap are typically recommended for regular hygiene.

29. Correct answer:

D. A probe will be introduced into the rectum. A prostate ultrasound, also known as a transrectal ultrasound (TRUS), is typically conducted by inserting a small ultrasound probe into the rectum. This probe generates sound waves which bounce off the prostate to create an image. This allows the doctor to visualize the prostate gland and assess its size and condition. The procedure is generally safe, with minimal discomfort, and can help diagnose conditions such as prostate cancer or benign prostatic hyperplasia.

Incorrect answer options:

A. The procedure involves the use of x-rays to generate a visual image. This is incorrect because ultrasound, not x-ray, is used to generate an image of the prostate. Ultrasounds use high-frequency sound waves, not radiation, to create images.

B. The procedure involves a large amount of radiation exposure. This is incorrect because ultrasound technology does not involve radiation. Instead, it uses sound waves to create images, which is generally considered safe and carries minimal risk.

C. The procedure is conducted using a cystoscope. This is incorrect. A cystoscope is used for procedures involving the bladder, not the prostate. For a prostate ultrasound, an ultrasound probe is used, which is inserted into the rectum to create an image of the prostate.

30. Correct answer:

A. Following a warm bath or shower. The most appropriate time to perform a testicular self-examination is after a warm bath or shower. The warmth relaxes the scrotal skin, making the testicles easier to examine and any abnormalities easier to detect.

Incorrect answer options:

B. After physical exercise. This is not the best time to perform a testicular self-examination. Exercise may cause temporary changes to the scrotum and testicles, such as contraction and elevation, which could make the examination more difficult and potentially lead to incorrect conclusions.

C. Right after rising from bed in the morning. Upon waking, the scrotal skin may not be relaxed enough for a thorough examination. As such, this is not typically recommended as the optimal time for testicular self-examination.

D. Just before going to bed at night. While it is certainly possible to perform a testicular self-examination at this time, it’s generally easier and more effective after a warm bath or shower, which relaxes the scrotal skin, making the testicles easier to examine.

31. Correct answer:

A. A certain amount of live sperm may remain in the ejaculatory fluid for a while. After a vasectomy, it’s possible for some sperm to remain in the upper part of the vas deferens, beyond the point where the tube is sealed off. This means that a man could still potentially fertilize an egg for a while after the procedure. It is usually recommended that another form of birth control is used until a semen analysis shows no sperm are present, which is typically checked a few months after the procedure.

Incorrect answer options:

B. Despite having undergone a vasectomy, it’s still recommended to use a condom for 1 to 2 years. This is not entirely accurate. While it is recommended to use another form of birth control until a semen analysis shows no sperm, this typically occurs within a few months, not 1 to 2 years.

C. The sperm count will not register as negative until his testosterone level decreases. This is incorrect. A vasectomy does not affect testosterone levels. The procedure simply blocks the path that sperm take from the testes (where they are produced) to the urethra.

D. Vasectomy doesn’t impact fertility immediately; it takes a few weeks for the body to adjust. While it’s true that a vasectomy doesn’t immediately prevent pregnancy, it’s not because the body needs time to adjust. Rather, it’s due to the presence of sperm already beyond the point of the vasectomy in the vas deferens.

32. Correct answer:

B. Has a high incidence of metastasis at early stages. Testicular cancer, though relatively rare compared to other types of cancer, can indeed metastasize, or spread, quickly. It can spread to the lymph nodes, lungs, liver, bones, and other parts of the body. That’s why early detection through self-examination and prompt medical evaluation of any abnormalities are important.

Incorrect answer options:

A. Is more common in older men above the age of 60. This is incorrect. Testicular cancer most commonly affects younger men, typically between the ages of 15 and 35. It’s one of the most common cancers in this age group, though it can occur at any age.

C. Is not identifiable through laboratory tests. This is incorrect. While testicular self-examination is important for early detection, testicular cancer can also be identified through a variety of laboratory tests, such as tumor marker tests, and imaging studies, such as ultrasound.

D. Seldom metastasizes. This is incorrect. Testicular cancer can metastasize, especially if not detected and treated early. That’s why regular testicular self-examinations are important for early detection.

33. Correct answer:

C. The urethra and the bladder. A cystectomy is a surgical procedure to remove all or part of the bladder. Depending on the specifics of the procedure, the surgeon may also visualize and potentially operate on the urethra, given its close anatomical relationship with the bladder.

Incorrect answer options:

A. The bladder and the kidneys. While the bladder is the primary focus during a cystectomy, the kidneys are not typically visualized during this procedure. The kidneys are located higher in the abdominal cavity and would not be directly accessed or visualized during a cystectomy unless there was a specific indication to do so.

B. The kidneys and the ureters. The kidneys and ureters are not the primary focus during a cystectomy. While the ureters, which connect the kidneys to the bladder, may be involved depending on the extent of the disease and the specific surgical procedure, the kidneys themselves would not typically be visualized during a cystectomy.

D. The prostate and the ureters. While the prostate may be involved in a radical cystectomy in men (due to its close anatomical relationship with the bladder), it’s not a primary focus in all cystectomies. Ureters may be visualized or modified during the procedure if urinary diversion is required, but they are not the main focus of a cystectomy.

34. Correct answer:

C. Blood in the urine without any pain. One of the most common signs of bladder cancer is painless hematuria, or blood in the urine. The blood can be visible to the naked eye (gross hematuria) or may only be detectable with a microscope (microscopic hematuria). While other symptoms can be associated with bladder cancer, painless hematuria is often the first sign.

Incorrect answer options:

A. Pain above the pubic bone. While pain above the pubic bone can be a symptom of various urinary conditions, it’s not a primary symptom associated with bladder cancer. It’s more commonly associated with conditions like urinary tract infections or interstitial cystitis.

B. Painful urination. Painful urination, or dysuria, can also be a symptom of bladder cancer, but it’s more commonly associated with other conditions like urinary tract infections, bladder stones, or interstitial cystitis. It’s not typically the primary symptom of bladder cancer.

D. Inability to completely empty the bladder. While an inability to completely empty the bladder can be associated with several urinary conditions, it’s not a primary symptom associated with bladder cancer. This symptom is more commonly seen with conditions such as benign prostatic hyperplasia (BPH), bladder stones, or certain neurological conditions that affect bladder function.

35. Correct answer:

A. The problem may necessitate surgical intervention. A hydrocele is an accumulation of fluid in the sac around the testicle, leading to swelling. It’s often painless and may not require treatment if it’s small and doesn’t cause discomfort. However, if it’s large or causing discomfort, surgical intervention might be necessary. This typically involves a procedure known as a hydrocelectomy to drain the fluid and possibly seal the sac to prevent fluid from accumulating again.

Incorrect answer options:

B. The issue typically requires extensive medical or surgical intervention. This statement is generally not accurate. While a hydrocele may require surgical intervention, this is usually a relatively straightforward procedure, and it’s not considered extensive intervention. A hydrocele often doesn’t cause serious complications, and many cases don’t require any treatment at all.

C. Medication is usually effective in controlling fluid accumulation. This is incorrect. Medications are not typically used to treat a hydrocele. In some cases, a doctor might recommend draining the fluid with a needle, a procedure known as aspiration, but this is less common because the fluid often reaccumulates after this procedure.

D. Regular exercise and a healthy diet typically resolve the issue. This is incorrect. While a healthy lifestyle is generally beneficial, it won’t specifically address or resolve a hydrocele. The fluid accumulation is not linked to diet or exercise habits.

36. Correct answer:

A. The initial symptoms of ovarian cancer tend to be unclear. Ovarian cancer often doesn’t cause any specific symptoms until it has spread beyond the ovary. When symptoms do occur, they are often vague and nonspecific, such as abdominal bloating, discomfort, or changes in bowel or bladder habits. This can make ovarian cancer difficult to detect in its early stages.

Incorrect answer options:

B. An irregular menstrual cycle is the most noticeable early symptom. An irregular menstrual cycle is not typically a symptom of ovarian cancer. Symptoms of ovarian cancer are often nonspecific and may include abdominal bloating, discomfort, feeling full quickly when eating, or changes in bowel or bladder habits.

C. Ovarian cancer only affects women after menopause. While the risk of ovarian cancer does increase with age and is highest in women who have gone through menopause, it can affect women of any age. There are also certain types of ovarian cancer, like germ cell tumors, that are more common in younger women.

D. Chemotherapy isn’t employed in the treatment of ovarian cancer. This is incorrect. Chemotherapy is a standard treatment for ovarian cancer, often used after surgery to kill any remaining cancer cells. In some cases, it may be used before surgery to shrink tumors.

37. Correct answer:

D. Severe itching in the vaginal and perineal areas. Candida albicans is a type of yeast that can cause an infection known as candidiasis or yeast infection when it overgrows. One of the most common symptoms of Candida-induced vaginitis (vaginal yeast infection) is intense itching in the vaginal and perineal areas. Other symptoms can include burning, redness, swelling, and a thick, white, cottage cheese-like vaginal discharge.

Incorrect answer options:

A. Sudden weight gain. This symptom is not typically associated with Candida-induced vaginitis. Weight gain can be associated with many conditions, but it’s not typically a sign of a yeast infection.

B. Upper abdominal discomfort. While abdominal discomfort can be a symptom of many conditions, it’s not typically associated with Candida-induced vaginitis. Abdominal discomfort is more commonly associated with conditions affecting the digestive system or other parts of the abdominal cavity.

C. Reduced urinary output. Reduced urinary output is not typically associated with Candida-induced vaginitis. This symptom is more commonly associated with urinary tract conditions or systemic conditions affecting fluid balance, like heart failure or kidney disease.

38. Correct answer:

D. Descending from the pubic area towards the anus. When performing perineal care, the nurse should cleanse from the least contaminated area to the most contaminated area to prevent the spread of microorganisms from the anal region to the urethral orifice. This means moving the washcloth or cotton ball from the pubic area (front) towards the anus (back).

Incorrect answer options:

A. Ascending from the anal region towards the pubic area. This is incorrect because it would spread microorganisms from the more contaminated anal area to the less contaminated pubic area, increasing the risk of infection.

B. Zigzagging from the left to the right across the labia majora. This is also incorrect as it doesn’t follow the principle of cleansing from the least contaminated to the most contaminated area. In addition, it could potentially spread microorganisms from one side of the perineal area to the other.

C. In a side-to-side motion across the labia majora. This method is also incorrect for the same reasons as option B. It doesn’t follow the principle of cleansing from the least contaminated to the most contaminated area and could spread microorganisms across the perineal area.

39. Correct answer:

A. Request that the client urinates prior to the examination. Asking a patient to empty their bladder before a pelvic exam can make the examination more comfortable. It can also provide a clearer view of the pelvic organs during the examination.

Incorrect answer options:

B. Instruct the client to maintain shallow breathing throughout the examination. This is not correct. During a pelvic exam, it can actually be helpful for the patient to take slow, deep breaths. This can help the muscles in the pelvic area to relax, which can make the examination more comfortable and allow the healthcare provider to get a more accurate examination.

C. Inform the client that anesthesia will be administered for the examination. This is not correct. Anesthesia is not typically used during a routine pelvic exam. While the examination can cause some discomfort, it is not usually painful enough to require anesthesia.

D. Assure the client that the examination will cause no discomfort. This is not entirely correct. While a pelvic exam should not cause severe pain, it can cause some discomfort. Each person’s experience can vary, and it’s important to be honest with patients about what they may experience.

40. Correct answer:

A. To identify cervical cancer. The primary purpose of a Pap test (also known as a Pap smear) is to detect abnormal cells in the cervix that may indicate cervical cancer or precancerous changes. Regular Pap tests can help catch these changes early, when treatment is most effective.

Incorrect answer options:

B. To diagnose ovarian cysts. Pap tests do not diagnose ovarian cysts. Ovarian cysts are typically diagnosed through physical examination, ultrasound, or other imaging studies.

C. To detect infections in the uterus. While a Pap test can sometimes detect infections like Human Papillomavirus (HPV) that can lead to cervical cancer, it is not primarily used to detect uterine infections. Other tests, such as a culture or biopsy, are typically used to diagnose infections in the uterus.

D. To confirm early pregnancy. A Pap test is not used to confirm pregnancy. Pregnancy is typically confirmed through a blood or urine test that detects the presence of the hormone human chorionic gonadotropin (hCG).

41. Correct answer:

C. Patient lies on their back with legs bent, hips rotated outward, and feet in stirrups (Lithotomy position). The lithotomy position is commonly used for vaginal examinations. This position provides good visibility and access to the pelvic area, which is crucial for a comprehensive examination.

Incorrect answer options:

A. Patient lies on their back with legs bent and feet flat (Dorsal recumbent position). The dorsal recumbent position is sometimes used for abdominal examinations or in situations where the lithotomy position is not suitable or comfortable for the patient. However, the lithotomy position is typically preferred for vaginal examinations due to the better visibility and access it provides.

B. Patient lies on their left side with right knee bent and raised towards the chest (Sim’s position). The Sim’s position is used for certain procedures such as rectal or sigmoid examinations, enema administration, or assessing the posterior aspect of the body. It is typically not used for vaginal examinations.

D. Patient lies on their stomach with arms at the side (Prone position). The prone position, with the patient lying on their stomach, is not suitable for a vaginal examination as it does not provide access or visibility to the pelvic area.

42. Correct answer:

A. Refrain from straining and lifting heavy objects until approved by the physician. After a procedure such as electrocauterization of the cervix, it’s important to avoid activities that could strain or disrupt the treated area, including lifting heavy objects. This can help prevent complications such as bleeding or damage to the cervix.

Incorrect answer options:

B. Rest in bed for the following five days. While rest is important after a medical procedure, it’s not typically necessary to remain in bed for five days after electrocauterization of the cervix. The patient should gradually resume normal activities as tolerated and advised by the healthcare provider.

C. Use a douche the next day to clear out debris and blood clots. Douching after this procedure is not recommended. It can disrupt the healing process and potentially introduce bacteria into the vagina, increasing the risk of infection.

D. Consume a diet high in spicy foods to promote healing. There is no evidence that a diet high in spicy foods promotes healing after electrocauterization of the cervix. A balanced diet, rich in proteins, vitamins, and minerals can support overall health and recovery, but specific dietary recommendations should be personalized based on the patient’s health status and preferences.

43. Correct answer:

A. Directs heat to the legs, altering the intended effect of heat applied to the pelvic region. A sitz bath is designed to direct warm water to the perineal and lower abdominal area. The targeted application of heat promotes blood flow, reduces inflammation, and aids in the healing process. A regular bathtub, by immersing the entire lower body including the legs, does not provide the same localized effect.

Incorrect answer options:

B. Is unable to provide water at the optimal temperature required for this procedure. While it’s crucial to ensure the water is at a comfortable and therapeutic temperature for a sitz bath, a regular bathtub can technically hold water at the same temperature. However, maintaining the optimal temperature might be more challenging over time in a regular bathtub.

C. May cause an allergic reaction due to the commonly used cleaning products. While it’s important to ensure a clean environment for any therapeutic procedure, a regular bathtub does not inherently pose a higher risk of allergic reactions. Cleaning products should be thoroughly rinsed off before any therapeutic use of a bathtub.

D. Does not maintain the same level of cleanliness as a specialized sitz bath tub. Both a regular bathtub and a sitz bath tub need to be cleaned appropriately before use. The cleanliness depends on proper cleaning and maintenance, not the type of bathtub.

44. Correct answer:

C. Hydrogen peroxide. Hydrogen peroxide is commonly used to clean the inner cannula of a tracheostomy tube because it has antimicrobial properties and can effectively remove mucus and other debris. After cleaning with hydrogen peroxide, the cannula should be rinsed thoroughly with sterile saline to remove any residual hydrogen peroxide, as it can cause tissue irritation if left in contact with the mucous membranes.

Incorrect answer options:

A. A solution of povidone-iodine. While povidone-iodine has antiseptic properties, it is not typically used to clean the inner cannula of a tracheostomy tube. It can cause staining and isn’t as effective at removing mucus and debris as hydrogen peroxide.

B. A solution of baking soda. Baking soda is not typically used in medical settings for cleaning medical equipment. It does not have the same antimicrobial properties as hydrogen peroxide.

D. Sodium hydrochloride. Sodium hydrochloride (also known as sodium hypochlorite or bleach) is too harsh to be used for cleaning a tracheostomy tube. It can cause corrosion of the tube and irritation to the patient’s airway.

45. Correct answer:

A. Assist the patient in changing positions to alleviate discomfort. In case of a swollen knee causing discomfort, it can be helpful to assist the patient in changing positions. This may relieve pressure on the knee and help reduce swelling. Additionally, elevating the affected leg can also help decrease swelling.

Incorrect answer options:

B. Promote exercises that engage the quadriceps muscles. While exercises that engage the quadriceps muscles can be beneficial for knee health, they may not be appropriate when the knee is actively swollen and causing discomfort. Overexertion could potentially exacerbate the condition.

C. Encourage the patient to bend the knee repeatedly. Repeated bending of a swollen, painful knee could potentially worsen the condition. Gentle movement may be beneficial, but repetitive bending could lead to further discomfort and damage.

D. Conduct passive range of motion exercises during each shift. While passive range of motion exercises can be beneficial in maintaining joint mobility, they should be performed with caution when a joint is swollen and causing discomfort. Overdoing such exercises could potentially exacerbate the condition.

46. Correct answer:

C. Gently insert the applicator tip in an upward and forward motion. When inserting a vaginal suppository, the nurse should gently insert the applicator tip in an upward and forward motion. This direction aligns with the natural curve of the vagina and can help ensure that the medication is delivered effectively.

Incorrect answer options:

A. Apply petroleum jelly to the tip of the suppository. Applying petroleum jelly to the tip of the suppository is not typically recommended. The suppository itself is designed to melt at body temperature and does not require an additional lubricant. However, some patients might benefit from a small amount of water-based lubricant on the applicator for comfort during insertion.

B. Insert the applicator about ½ inch deep and then press the plunger. Inserting the applicator only ½ inch deep may not allow for effective delivery of the medication. The applicator should typically be inserted as far as it will comfortably go, or as instructed by the healthcare provider or manufacturer’s instructions.

D. Apply alcohol to the suppository before insertion. Alcohol should not be applied to the suppository before insertion. Alcohol can cause irritation to the sensitive tissues of the vagina.

47. Correct answer:

A. Knee swelling. A stationary thrombus, or a blood clot that remains in place without moving, can cause local symptoms such as swelling, redness, and warmth, especially if it forms in a vein in the leg or knee.

Incorrect answer options:

B. Severe headache. While a severe headache could potentially be a symptom of a thrombus or blood clot in the brain, it is less likely to be associated with a thrombus in other parts of the body.

C. Frequent hiccups. Frequent hiccups are not typically associated with the formation of a thrombus. They are more commonly related to conditions affecting the diaphragm or stomach, neurological conditions, or certain medications.

D. Foot numbness. While foot numbness could potentially be a symptom of a thrombus or blood clot in the leg, it is not as typical or as likely as swelling, redness, or warmth.

48. Correct answer:

A. Wrap the moist gauze in a towel. When applying a warm moist compress, the moist gauze should be wrapped in a towel or cloth before applying it to the skin. This prevents direct contact of the moist heat with the skin, which could cause burns or irritation. It also helps to maintain the warmth of the compress.

Incorrect answer options:

B. Maintain a sterile technique. While maintaining cleanliness is important when applying a compress, it is not necessary to maintain a sterile technique. This type of treatment is considered clean rather than sterile.

C. Heat the water to 120°F. Water heated to 120°F is too hot for direct skin contact and could cause burns. The water used for a warm compress should be warm to touch but not uncomfortably hot.

D. Keep the compress on for a continuous 24 hours. A warm compress should not be left on continuously for 24 hours. It should be applied for a period of time as recommended by a healthcare provider, typically 15-20 minutes, and then removed to allow the skin to rest.

49. Correct answer:

D. Envelop them in a warm blanket. To warm cold feet, it is appropriate to cover them with a warm blanket. This allows for gentle, safe warming.

Incorrect answer options:

A. Use an electric heating pad. Using an electric heating pad can be risky, particularly for patients with certain health conditions like diabetes or vascular disease who may have reduced sensation in their feet. They may not be able to accurately feel the temperature, which could lead to burns.

B. Place her feet in a basin of cold water. Placing cold feet in cold water will not help to warm them, and it could potentially make them colder.

C. Elevate her feet on a stool. Elevating the feet does not inherently provide warmth. While it can help with swelling, it is not the best action for warming cold feet.

50. Correct answer:

A. Red patches covered with silvery scales. Psoriasis is a chronic skin condition characterized by the rapid build-up of skin cells leading to red patches covered with silvery scales. These patches can be itchy or painful and are often found on the scalp, knees, elbows, and lower back, but can appear anywhere on the body.

Incorrect answer options:

B. Oozing lesions on the body’s trunk. This description may be more indicative of conditions such as impetigo, a bacterial skin infection, or possibly a severe case of eczema, rather than psoriasis.

C. A rash marked by raised, pus-filled lesions. This could describe a skin infection, such as folliculitis or possibly a condition like acne, but not typically psoriasis. There is a type of psoriasis, called pustular psoriasis, that involves white blisters of noninfectious pus (consisting of white blood cells), but it is less common and the pus-filled lesions aren’t generally described as a “rash”.

D. Dark, rough patches on the hands and feet. This could describe a variety of skin conditions, including calluses or a condition known as keratoderma, but it does not typically describe psoriasis.